0% found this document useful (0 votes)
12 views4 pages

Solhw 1

Chapter 12 provides solutions to various exercises related to prime numbers, even and odd integers, and properties of rational and irrational numbers. It includes proofs and computations demonstrating concepts such as modular arithmetic, closure properties of sets under specific operations, and the divisibility of certain expressions. The chapter emphasizes the importance of logical reasoning and contradiction in mathematical proofs.

Uploaded by

timeofdeath321
Copyright
© © All Rights Reserved
We take content rights seriously. If you suspect this is your content, claim it here.
Available Formats
Download as PDF, TXT or read online on Scribd
0% found this document useful (0 votes)
12 views4 pages

Solhw 1

Chapter 12 provides solutions to various exercises related to prime numbers, even and odd integers, and properties of rational and irrational numbers. It includes proofs and computations demonstrating concepts such as modular arithmetic, closure properties of sets under specific operations, and the divisibility of certain expressions. The chapter emphasizes the importance of logical reasoning and contradiction in mathematical proofs.

Uploaded by

timeofdeath321
Copyright
© © All Rights Reserved
We take content rights seriously. If you suspect this is your content, claim it here.
Available Formats
Download as PDF, TXT or read online on Scribd
You are on page 1/ 4

Chapter 12

Solutions to the Exercises

“Intuition comes from experience, experience from failure, and


failure from trying.”

Exercises for Chapter 1


Exercise 1. Show that 2 is the only prime number which is even.

Solution. Take p a prime number. Then p has only 2 divisors, 1 and p. If p


is also even, then one of its divisors has to be 2, thus p = 2.

Exercise 2. Show that if n2 is even, then n is even, for n an integer.

Solution. An integer n is either even, that is n = 2n0 , for some integer n0 ,


or odd, that is n = 2n0 + 1 for some integer n0 . Thus n2 is either 4(n0 )2 or
4(n0 )2 + 4n0 + 1. The case where n2 is even is thus when n = 2n0 .

Exercise 3. The goal of this exercise is to show that 2 is irrational. We
provide a step by step way of doing so.
√ √
1. Suppose by contradiction that 2 is rational, that is 2 = m
n
, for m
and n integers with no common factor. Show that m has to be even,
that is m = 2k.

2. Compute m2 , and deduce that n has to be even too, a contradiction.

267
268 CHAPTER 12. SOLUTIONS TO THE EXERCISES
√ √ m
Solution. 1. Suppose by contradiction that 2 is rational, that is 2 = n
,
for m and n integers with no common factor. Then
m2
2=
n2
and thus m2 = 2n2 , showing that m2 is even, that is, using Exercise 2,
m has to be even, say m = 2k for k some integer.
2. Now m2 = (2k)2 = 4k 2 . This tells us, combining with the first step of
the exercise, that
m2 = 4k 2 = 2n2
which implies that 2k 2 = n2 , that is n2 is even and by again by Exercise
2, it must be that n is even. This is a contradiction, since we assumed
that m and n have no commun factor.
Exercise 4. Let n be an integer greater than 1. Suppose that a mod n = a0
and b mod n = b0 . Show that
1. (a + b) mod n ≡ (a0 + b0 ) mod n,
2. (a · b) mod n ≡ (a0 · b0 ) mod n.
Solution. 1. Since (a mod n) = a0 , then a = qn+a0 , and since (b mod n) =
b , then similarly b = rn + b0 , for some integers q, r. Then
0

(a + b) mod n = (qn + a0 + rn + b0 ) mod n ≡ (a0 + b0 ) mod n.

2. Similarly
(a·b) mod n ≡ (qn+a0 )(rn+b0 ) ≡ qrn2 +qnb0 +a0 rn+a0 b0 mod n ≡ (a0 b0 ) mod n.
Exercise 5. Compute the addition table and the multiplication tables for
integers modulo 4.
Solution. We represent integers modulo 4 by the set of integers {0, 1, 2, 3}.
Then

+ 0 1 2 3
0 0 1 2 3
1 1 2 3 0
2 2 3 0 1
3 3 0 1 2
269

Similarly

* 0 1 2 3
0 0 0 0 0
1 0 1 2 3
2 0 2 0 2
3 0 3 2 1
Note that these tables are great to observe the closure property! Elements
computed in these tables are the same as those given as input.
m(m+1)
Exercise 6. Show that 2
≡ 0 (mod m) for m an odd number.

Solution. Suppose that m is an odd number. Then m + 1 is even, thus


divisible by 2, say m + 1 = 2k for some k. Now

m(m + 1)
= mk ≡ 0 (mod m).
2
You may also observe that it is not always true for even numbers. If for
example m = 2, this does not work, indeed 2 · 3/2 = 3 which is not 0
mod 2.

Exercise 7. 1. Compute 7 · 8 · 9 · 10 modulo 3.

2. Show that n3 − n is always divisible by 3, for n any positive integer.

Solution. 1. Since 3 divides 9, the result modulo 3 is 0.

2. We note that n3 − n = n(n2 − 1) = n(n − 1)(n + 1). Now any positive


integer n is either a multiple of 3, say n = 3k, or when divided by 3
there is a remainder of 1, say n = 3k+1, or a remainder of 2, n = 3k+2.
If n = 3k, n3 − n = 3k(n − 1)(n + 1) is divisible by 3, if n = 3k + 1
then n3 − n = n(3k)(n + 1) is divisible by 3 and if n = 3k + 2, then
n3 − n = n(n − 1)(3k + 3) is divisible by 3.
This can be rewritten by considering integers modulo 3, this is the same
idea. To show that 3 divides n3 − n is the same thing has n3 − n ≡ 0
(mod 3). Then once one has the idea to look at integers modulo 3,
write n as 3k, 3k + 1, or 3k + 2, and compute n3 − n for each case, for
example (3k)3 − (3k) is clearly divisible by 3, the same computation
can be done to show that n3 − n is a multiple of 3 for 3k + 1 and 3k + 2.
270 CHAPTER 12. SOLUTIONS TO THE EXERCISES

Exercise 8. Consider the following sets S, with respective operator ∆.

• Let S be the set of rational numbers, and ∆ be the multiplication. Is


S closed under ∆? Justify your answer.

• Let S be the set of natural numbers, and ∆ be the subtraction. Is S


closed under ∆? Justify your answer.

• Let S be the set of irrational numbers, and ∆ be the addition. Is S


closed under ∆? Justify your answer.

m m0
Solution. • Take two rational numbers n
and n0
. Then

m m0 mm0
n n0
= nn0

which is a rational number. Thus the answer is yes, S is closed under


multiplication.

• The subtraction of two natural numbers does not always give a number
natural, for example,
5 − 10 = −5.

Thus S is not closed under subtraction.

• The addition of two irrational numbers does not always give an irra-
tional number, for example,
√ √
(2 + 2) + (2 − 2)) = 4

and 4 is not an irrational number. Thus S is not


√ closed under addition.
Note that we are using
√ here the claim that 2 + √ 2 is irrational. Indeed,
suppose that 2 + 2 were rational, that is 2 + 2 = m n
for m, n some
integers. Then
√ m m − 2n
2= −2=
n n

which is a contradiction to the fact that 2 is irrational.

You might also like